Đến nội dung

LNH

LNH

Đăng ký: 16-12-2012
Offline Đăng nhập: Riêng tư
****-

#695518 Chuyện về những người ăn học không đến nơi đến chốn - bb1412 và vth

Gửi bởi LNH trong 26-10-2017 - 08:16

.




#695291 Chuyện về những người ăn học không đến nơi đến chốn - bb1412 và vth

Gửi bởi LNH trong 23-10-2017 - 20:05




#588849 Đăng ký tham gia dự thi VMEO IV

Gửi bởi LNH trong 14-09-2015 - 12:38

Họ tên: Lê Nhật Hoàng
Nick trong diễn đàn (nếu có): LNH
Năm sinh: 1998
Dự thi cấp: THPT



#571755 Kinh hoàng sự thật đằng sau BQT VMF và cái giá của 1 ĐHV Tổng hợp

Gửi bởi LNH trong 12-07-2015 - 15:27

Xem xong bài viết này em quyết định chi 10 tỷ vì chức vụ ĐHV Tổng hợp :))




#560227 $n\sqrt{d}\left \{ n\sqrt{d...

Gửi bởi LNH trong 18-05-2015 - 20:55

$\boxed{\text{Problem 2}}$ (Balkan MO 2015)

Chứng minh rằng trong $20$ số nguyên dương liên tiếp có một số nguyên dương $d$ sao cho với mọi số nguyên dương $n$, bất đẳng thức sau đúng:

$n\sqrt{d}\left \{ n\sqrt{d} \right \}>\frac{5}{2}$




#544859 Chứng minh rằng: $\left | S_1 \right |-\left | S_2 \...

Gửi bởi LNH trong 19-02-2015 - 00:00

Cho $k$ là số nguyên dương chẵn. $N$ là tích của $k$ số nguyên tố phân biệt $p_1,...,p_k$.  $a,b$ là hai số nguyên dương phân biệt sao cho $a \leq b \leq N$. Gọi $S_1$ và $S_2$ là hai tập thỏa mãn:$ S_1=\{d| $ $ d|N, a\leq d\leq b, d $ có số ước nguyên tố chẵn $\}$, $ S_2=\{d| $ $ d|N, a\leq d\leq b, d $ có số ước nguyên tố lẻ $\}$. Chứng minh rằng: $\left | S_1 \right |-\left | S_2 \right |\leq C_{k}^{\frac{k}{2}}$




#542841 Thảo luận về DS dự thi và KQ thi VMO 2015

Gửi bởi LNH trong 03-02-2015 - 16:56

Theo em được biết thì VMF mình có 2 Hiệp sĩ là Cao Xuân Huy và nguyenta98 đạt giải nhì

P/s: em cũng vậy (khoe khoang tí :P )




#542058 Chứng minh có tồn tại $f(\{a,b\})=f(\{b,c...

Gửi bởi LNH trong 27-01-2015 - 17:31

$n$ là nguyên dương và $|S|=2^n+1$.Cho $f$ là hàm từ tập các tập con hai phần tử của $S$ tới $\{0,1,...,2^{n-1}-1\}$ sao cho với mỗi $f(\{x,y\}),f(\{y,z\}),f(\{z,x\})$ sẽ bằng tổng hai số còn lại.Chứng minh có tồn tại $f(\{a,b\})=f(\{b,c\})=f(\{c,a\})=0$.

 

Ta chứng minh bằng quy nạp.

Với $n=1$ thì bài toán hiển nhiên đúng.

GIả sử bài toán đúng với $n=k$, $k \geq 1$

Xét $n=k+1$:

Xét $a \in S$, ta quy ước $f\left ( \left \{ a,a \right \} \right )=0$

Ta phân hoạch $S$ thành $2$ tập $U,V$ như sau:

Với $x \in S$, $x \in U$ kvck $f\left ( \left \{ x,a \right \} \right )$ chẵn

Với $x \in S$, $x \in V$ kvck $f\left ( \left \{ x,a \right \} \right )$ lẻ

Theo nguyên tắc Dirichlet, tồn tại một trong $2$ tập $U,V$ có số phần tử không nhỏ hơn $2^k+1$

Giả sử $\left | U \right | \geq 2^k+1$

Với mọi x,y thuộc $U$, ta có $f\left ( \left \{ x,a \right \} \right )+f\left ( \left \{ y,a \right \} \right )+f\left ( \left \{ x,y \right \} \right ) \vdots 2$

$f\left ( \left \{ x,a \right \} \right )+f\left ( \left \{ y,a \right \} \right ) \vdots 2$

Suy ra $f\left ( \left \{ x,y \right \} \right ) \vdots 2, \forall x,y \in U$

Đặt $g\left ( \left \{ x,y \right \} \right )=\frac{f\left ( \left \{ x,y \right \} \right )}{2}$ $\Rightarrow g\left ( \left \{ x,y \right \} \right ) \in \left \{ 0,...,2^{k-1}-1 \right \}$

Theo giả thiết quy nạp, tồn tại $x,y,z \in U$ sao cho $g\left ( \left \{ x,y \right \} \right )=g\left ( \left \{ x,z \right \} \right )=g\left ( \left \{ y,z \right \} \right )=0$

Suy ra  $f\left ( \left \{ x,y \right \} \right )=f\left ( \left \{ x,z \right \} \right )=f\left ( \left \{ y,z \right \} \right )=0$

Theo nguyên lí quy nạp, ta có đpcm




#535519 Topic ôn luyện VMO 2015

Gửi bởi LNH trong 30-11-2014 - 12:05

Bài 40 : Tìm hàm $f$ liên tục $\mathbb{R}\to \mathbb{R}$ sao cho $f(x)-f(y)\in \mathbb{Q}\forall x-y \in \mathbb{Q}$

Đặt $g(x)=f(x)-f(0)$, ta có $g(0)=0$ và $g(x)-g(y) \in \mathbb{Q}\Leftrightarrow x-y \in \mathbb{Q}$

Cố định số hữu tỉ $r$, xét $h(x)=g(x+r)-g(x)$

Ta có $h(x) \in \mathbb{Q}, \forall x \in\mathbb{R}$

Mà $h$ liên tục trên $\mathbb{R}$ nên $h(x)=const$

$h\left ( 0 \right )=g\left ( r \right )\Rightarrow g\left ( x+r \right )=g\left ( x \right )+g\left ( r \right ), \forall r \in \mathbb{Q}$

Từ đây, ta chứng minh được $g\left ( x \right )=xg(1), \forall x \in \mathbb{Q}$

Mà $g$ liên tục trên $\mathbb{R}$ nên $g\left ( x \right )=xg\left ( 1 \right ),\forall x \in \mathbb{R}$

Suy ra $f\left ( x \right )\equiv ax+b$ với $a,b \in \mathbb{Q}$ (thử lại thấy thỏa mãn)




#534768 Topic ôn luyện VMO 2015

Gửi bởi LNH trong 25-11-2014 - 20:54

Bài 46: Cho tập $S$ gồm $1953$ điểm thỏa mãn $2$ điểm bất kì của $S$ cách nhau ít nhất $1$ cm. Chứng minh rằng: tồn tại tập con của $S$ có $217$ điểm sao cho $2$ điểm bất kì trong tập con đều có khoảng cách ít nhất $\sqrt{3}$ cm.




#534442 Topic ôn luyện VMO 2015

Gửi bởi LNH trong 23-11-2014 - 19:45

Bài 41 : Cho tam giác $ABC$ nội tiếp đường tròn $(O)$. $H$ là hình chiếu của $A$ xuống $BC$, $AK$ là đường kính của $(O)$. $I$ và $J$ lần lượt là tâm đường tròn nội tiếp và bàng tiếp góc $A$ của $\Delta \,ABC$. Chứng minh rằng $\widehat{BIH}=\widehat{CIK}$ và $\widehat{BJH}=\widehat{CJK}$

Capture.PNG

Giả sử $AC > AB$

Gọi $D$ là giao điểm $AI$ và đường tròn tâm $O$

Ta cần chứng minh $\angle BIH=\angle CIK\Leftrightarrow \angle DIB-\angle DIC=\angle HID-\angle KID$

$\angle IBC-\angle ICB=\angle HID-\angle KID$

Gọi $T$ sao cho $TI \perp BI$ với $T \in AH$

Ta có: $\angle IBC=\angle HTI=\angle HAI+\angle TIA=\angle HAI+\angle ICB$

Vậy đpcm tương đương với:

$\angle HID=\angle HAI+\angle DIK$

$\Leftrightarrow \angle AHI=\angle DIK$ (1)

Ta có $I,B,C,J$ nằm trên đường tròn tâm $D$, $D$ là trung điểm của $IJ$

Mặt khác, $\angle ADK= 90^0$

Suy ra tam giác $IKJ$ cân tại $K$

Suy ra $\angle KID = \angle KJD$ (2)

$\angle HAI = \angle JAK$, $\frac{AH}{AI}=\frac{AJ}{AK}$ nên tam giác $AHI$ đồng dạng với tam giác $AJK$

Suy ra $ \angle AHI=\angle AJK$ (3)

Từ (1), (2), (3), ta có đpcm

Để chứng minh $\angle BJH=\angle CJK$, ta cần chứng minh:

$\angle JHI= \angle JKI$

$\Leftrightarrow IHC=\angle DKJ\Leftrightarrow \angle AHI=\angle DIK$ @~> đpcm




#534157 Topic ôn luyện VMO 2015

Gửi bởi LNH trong 22-11-2014 - 09:47

Bài 37: Cho các số nguyên $x,y,z$ với $x>2,y>1,z>0$ thỏa mãn:

$x^y+1=z^2$

Gọi $p$ là số ước nguyên tố phân biệt của $x$, $q$ là số ước nguyên tố phân biệt của $y$. Chứng minh rằng: $p \geq q+2$




#533793 Topic ôn luyện VMO 2015

Gửi bởi LNH trong 19-11-2014 - 18:47

Bài 35: Cho $2$ tập hợp $T$ và $P$, $T$ chứa $66$ điểm, $P$ chứa $16$ đường thẳng. $(A,l)$ được gọi là tốt nếu $A \in T$, $l \in P$, $A \in l$. Tìm số bộ tốt lớn nhất.




#533624 Topic ôn luyện VMO 2015

Gửi bởi LNH trong 17-11-2014 - 20:51

 

Bài 2: Tô tập số nguyên bởi $4$ màu. $x,y$ là số nguyên lẻ thỏa mãn $\left | x \right |$ khác $\left | y \right |$ . CMR: tồn tại 2 số nguyên cùng màu có hiệu thuộc ${x,y,x+y,x-y}$.

 

Giả sử phản chứng: tồn tại một cách tô tập $Z$, $f: \mathbb{Z}\rightarrow \left \{ X,D,T,V \right \}$ sao cho với mọi $a \in Z$, ta có:

$\left \{ f\left ( a \right ),f\left ( a+x \right ),f\left ( a+y \right ),f\left ( a+x+y \right ) \right \}=\left \{ X,D,T,V \right \}$

Xét $g:\mathbb{Z}^2\rightarrow \left \{ X,D,T,V \right \}$ sao cho $g\left ( i,j \right )=f\left ( ix+jy \right )$

Biểu diễn các điểm này trên mạng lưới nguyên

4 đỉnh của một hình vuông đơn vị đôi một khác màu

Xét một cột bất kì

Nếu tồn tại $3$ đỉnh liên tiếp trên cột khác màu thì cách tô trên t/m:

+)Mỗi hàng chỉ có $2$ màu

+)Nếu $2$ hàng cách nhau một số chẵn thứ tự thì cùng tập màu (*)

Nếu một cột bất kì không tồn tại $3$ ô liên tiếp khác màu khì (*) sẽ đúng cho các cột. Không mất tính tổng quát, giả sử (*) đúng.

Giả sử hàng $y=0$ được tô màu D,V

Nếu $g\left ( 0,0 \right )=D\Rightarrow g\left ( 0,y \right )=V,g\left ( x,0 \right )=V$

$\Rightarrow g\left ( 0,x \right )=f\left ( xy \right )=g\left ( y,0 \right )=V$

Mặt khác,  hàng chứa $\left ( 0,x \right )$ là hàng thứ $x$, hàng chứ $\left ( y,0 \right )$ là hàng thứ 0, suy ra vô lí

Vậy ta có đpcm




#532217 Topic ôn luyện VMO 2015

Gửi bởi LNH trong 07-11-2014 - 15:37

Hưởng ứng vài bài lí thuyết đồ thị :))

Bài 12: Cho một ngôi trường có $n$ khóa học và $n$ học sinh. Các học sinh đăng kí vào lớp học, không có $2$ học sinh nào tham gia các khóa học hoàn toàn giống nhau. CMR: ta có thể đóng cửa một khoá học sao cho vẫn không có hai học sinh nào tham gia các khoá học hoàn toàn going nhau.

Bài 13: Cho $35$ người đi dự một buổi họp. Có tất cả $112$ căp hai người quen nhau. Chứng minh rằng tồn tại $4$ người $a,b,c,d$ sao cho $a$ quen $b$, $b$ quen $c$, $c$ quen $d$ và $d$ quen $a$.